Bold Face CR

This topic has expert replies
Senior | Next Rank: 100 Posts
Posts: 31
Joined: Tue Sep 16, 2008 8:29 am
Location: Brazil
GMAT Score:460

OA

by robzoc » Mon Oct 27, 2008 6:53 am
Sorry for taking so long.

the OA from the GMAT PREP softwre is B

Master | Next Rank: 500 Posts
Posts: 347
Joined: Mon Aug 04, 2008 1:42 pm
Thanked: 1 times

Re: bold face in the wrong sentences...

by Stockmoose16 » Mon Oct 27, 2008 11:54 pm
gmat009 wrote:
robzoc wrote:Hey..

guys, look at the real GMAT PREP question...I noticed something was wrong. Luckly I have the question on file...
What's OA for this question.........
Obviously there are different versions of the question depending on what level you're performing at on the GMAT PREP. I think the original post is accurate, but I'm having a hard time deciding between A and C.

Newbie | Next Rank: 10 Posts
Posts: 3
Joined: Thu May 22, 2008 8:22 am

by ranmun » Wed Oct 29, 2008 4:28 pm
hi guys,
the correct answer is A.

Legendary Member
Posts: 645
Joined: Wed Sep 05, 2007 4:37 am
Location: India
Thanked: 34 times
Followed by:5 members

by camitava » Wed Oct 29, 2008 6:26 pm
Guys, why not C? I believe C is carrying all the points the passage is talking abt.
Correct me If I am wrong


Regards,

Amitava

Junior | Next Rank: 30 Posts
Posts: 28
Joined: Tue Oct 28, 2008 4:37 pm
Thanked: 2 times

by Somerandomguy » Wed Oct 29, 2008 7:34 pm
camitava wrote:Guys, why not C? I believe C is carrying all the points the passage is talking abt.
Statement 1: Pro-Tect cannot reduce the number of car-theft policies it carries, so cannot protect itself against continued large payouts that way.

Statement 2: Therefore, Pro-Tect has decided to offer a discount to holders of car-theft policies whose cars have antitheft devices.

(C) The first has been used as consideration to support adopting a certain strategy for achieving a goal; the second reports a decision to adopt an alternative strategy.


This is half wrong/right, respectively. The 1st statement is wrong because it eliminates an obvious alternative. It doesn't "support" anything but rather "opposes" an idea ("cannot protect itself against ..."). The 2nd statement is correct.

Junior | Next Rank: 30 Posts
Posts: 11
Joined: Fri Apr 02, 2010 7:35 am

by s9q78g » Sun Sep 05, 2010 9:40 am
During the past year, Pro-Tect Insurance Company's total payouts on car.theft claims were
larger than the company can afford to sustain. Pro-Tect cannot reduce the number of
car-theft policies it carries
, so cannot protect itself against continued large payouts that
way. Therefore, Pro-Tect has decided to offer a discount to holders of car-theft
policies whose cars have anti-theft devices.
Many policyholders will respond to the
discount by installing anti-theft devices, since the amount of the discount will within two years
typically more than cover the cost of installation. Thus, because cars with anti-theft devices are
rarely stolen, Pro-Tect's plan is likely to reduce its annual payouts.

In the argument above, the two portions in boldface play which of the following roles?

A. The first rules out a certain strategy for achieving a goal; the second presents the strategy that
was adopted instead and whose effectiveness the argument assesses.
B. The first is a judgment made in support of a certain conclusion; the second is that conclusion.
C. The first has been used as a consideration to support adopting a certain strategy for achieving
a goal; the second reports a decision to adopt an alternative strategy.
D. The first provides evidence in favor of adopting a certain strategy for achieving a goal; the
second reports a decision to pursue an alternative goal.
E. The first is a consideration offered against adopting a certain strategy for achieving a goal; the
second is the main conclusion that the argument is seeking to establish.
camitava wrote:
Guys, why not C? I believe C is carrying all the points the passage is talking abt.
Somerandomguy wrote:
Statement 1: Pro-Tect cannot reduce the number of car-theft policies it carries, so cannot protect itself against continued large payouts that way.

Statement 2: Therefore, Pro-Tect has decided to offer a discount to holders of car-theft policies whose cars have antitheft devices.

(C) The first has been used as consideration to support adopting a certain strategy for achieving a goal; the second reports a decision to adopt an alternative strategy.


This is half wrong/right, respectively. The 1st statement is wrong because it eliminates an obvious alternative. It doesn't "support" anything but rather "opposes" an idea ("cannot protect itself against ..."). The 2nd statement is correct.
Stuart- can you pls tell us how to attack such a question.
The OA is C
I am not clear about the statement- The first has been used as a consideration to support adopting a certain strategy for achieving a goal and
the second reports a decision to adopt an alternative strategy.
My question is- When the questioner mentions, "The certain strategy" in the first statement,
and "the Alternative Strategy" in the second statement,

does he mean that this certain strategy is the alternative strategy which the insurance company wants to adopt?

__________
Thank you

User avatar
GMAT Instructor
Posts: 3225
Joined: Tue Jan 08, 2008 2:40 pm
Location: Toronto
Thanked: 1710 times
Followed by:614 members
GMAT Score:800

by Stuart@KaplanGMAT » Tue Sep 07, 2010 12:08 pm
gmat009 wrote:During the past year, Pro-Tect Insurance Company's total payouts on car-theft claims were larger than the company can afford to sustain. Pro-Tect cannot reduce the number of car-theft policies it carries, so cannot protect itself against continued large payouts that way. Therefore, Pro-Tect has decided to offer a discount to holders of car-theft policies whose cars have antitheft devices. Many policyholders will respond to the discount by installing antitheft devices, since the amount of the discount will within two years typically more than cover the cost of installation. Thus, because cars with antitheft devices are rarely stolen, Pro-Tect's plan is likely to reduce its annual payouts.
In the argument above, the two portions in boldface play which of the following roles?
A. The first rules out a certain strategy for achieving a goal; the second presents the strategy that was adopted instead and whose effectiveness the argument assesses.
B. The first is a judgment made in support of a certain conclusion; the second is that conclusion.
C. The first has been used as a consideration to support adopting a certain strategy for achieving a goal; the second reports a decision to adopt an alternative strategy.
D. The first provides evidence in favor of adopting a certain strategy for achieving a goal; the second reports a decision to pursue an alternative goal.
E. The first is a consideration offered against adopting a certain strategy for achieving a goal; the second is the main conclusion that the argument is seeking to establish.

Plz. explain.....
Step 1 of the Kaplan Method for CR: Focus on the Question Stem

Before reading the stimulus, it's key to identify the question type; after all, different questions require different approaches.

Here, we identify a bolded statement question (one of the few question types you can identify just by looking at the stimulus). We think to ourselves: as I read, I need to abstract the argument - don't focus on the details, focus on the structure. I'll build a mini-roadmap of the stimulus and takes notes regarding the function of each sentence.

Step 2 of the Kaplan Method for CR: Untangle the Stimulus

5 sentences for our roadmap. Let's break it down as we go:

s1: fact about last year
b1: something the company CANNOT do
unbolded part of s2: result of b1
b2: what the company decided to do
s4: how policyholders will respond
s5: consequence of the plan

Step 3 of the Kaplan Method for CR: Make a Prediction

Since we've roadmapped b1 and b2, our prediction is ready to go!

First bolded statement: something the company cannot do
Second bolded statement: what the company decided to do

Step 4 of the Kaplan Method for CR: Attack the Choices

Since we've got a prediction, we scan aggressively looking for a match.

A) perfect!
B) no judgment - eliminate.
C) the first isn't support for the eventual strategy - eliminate.
D) the first isn't evidence - eliminate.
E) the second isn't a conclusion - eliminate.

For (C), no link is given between b1 and b2 - the author just states, matter of factly, that the first option isn't available. The fact that the first option isn't available is certainly what's led Pro-Tect to adopting a different approach, but isn't directly related to which approach Pro-Tect chose, which is what (C) implies.
Image

Stuart Kovinsky | Kaplan GMAT Faculty | Toronto

Kaplan Exclusive: The Official Test Day Experience | Ready to Take a Free Practice Test? | Kaplan/Beat the GMAT Member Discount
BTG100 for $100 off a full course

Senior | Next Rank: 100 Posts
Posts: 78
Joined: Wed May 16, 2012 8:57 pm
Thanked: 2 times

by divineacclivity » Sat May 31, 2014 7:27 pm
thank you for your explanation. I have a follow-up question on this one. Could you please tell me why's second not a conclusion (E). I was confused between options A & E.
I picked E thinking "Therefore post-tech has decided to ..." looked conclusion like a statement.
Stuart Kovinsky wrote:
gmat009 wrote:During the past year, Pro-Tect Insurance Company's total payouts on car-theft claims were larger than the company can afford to sustain. Pro-Tect cannot reduce the number of car-theft policies it carries, so cannot protect itself against continued large payouts that way. Therefore, Pro-Tect has decided to offer a discount to holders of car-theft policies whose cars have antitheft devices. Many policyholders will respond to the discount by installing antitheft devices, since the amount of the discount will within two years typically more than cover the cost of installation. Thus, because cars with antitheft devices are rarely stolen, Pro-Tect's plan is likely to reduce its annual payouts.
In the argument above, the two portions in boldface play which of the following roles?
A. The first rules out a certain strategy for achieving a goal; the second presents the strategy that was adopted instead and whose effectiveness the argument assesses.
B. The first is a judgment made in support of a certain conclusion; the second is that conclusion.
C. The first has been used as a consideration to support adopting a certain strategy for achieving a goal; the second reports a decision to adopt an alternative strategy.
D. The first provides evidence in favor of adopting a certain strategy for achieving a goal; the second reports a decision to pursue an alternative goal.
E. The first is a consideration offered against adopting a certain strategy for achieving a goal; the second is the main conclusion that the argument is seeking to establish.

Plz. explain.....
Step 1 of the Kaplan Method for CR: Focus on the Question Stem

Before reading the stimulus, it's key to identify the question type; after all, different questions require different approaches.

Here, we identify a bolded statement question (one of the few question types you can identify just by looking at the stimulus). We think to ourselves: as I read, I need to abstract the argument - don't focus on the details, focus on the structure. I'll build a mini-roadmap of the stimulus and takes notes regarding the function of each sentence.

Step 2 of the Kaplan Method for CR: Untangle the Stimulus

5 sentences for our roadmap. Let's break it down as we go:

s1: fact about last year
b1: something the company CANNOT do
unbolded part of s2: result of b1
b2: what the company decided to do
s4: how policyholders will respond
s5: consequence of the plan

Step 3 of the Kaplan Method for CR: Make a Prediction

Since we've roadmapped b1 and b2, our prediction is ready to go!

First bolded statement: something the company cannot do
Second bolded statement: what the company decided to do

Step 4 of the Kaplan Method for CR: Attack the Choices

Since we've got a prediction, we scan aggressively looking for a match.

A) perfect!
B) no judgment - eliminate.
C) the first isn't support for the eventual strategy - eliminate.
D) the first isn't evidence - eliminate.
E) the second isn't a conclusion - eliminate.

For (C), no link is given between b1 and b2 - the author just states, matter of factly, that the first option isn't available. The fact that the first option isn't available is certainly what's led Pro-Tect to adopting a different approach, but isn't directly related to which approach Pro-Tect chose, which is what (C) implies.